www.vorhilfe.de
- Förderverein -
Der Förderverein.

Gemeinnütziger Verein zur Finanzierung des Projekts Vorhilfe.de.
Hallo Gast!einloggen | registrieren ]
Startseite · Mitglieder · Impressum
Forenbaum
^ Forenbaum
Status VH e.V.
  Status Vereinsforum

Gezeigt werden alle Foren bis zur Tiefe 2

Navigation
 Startseite...
 Suchen
 Impressum
Das Projekt
Server und Internetanbindung werden durch Spenden finanziert.
Organisiert wird das Projekt von unserem Koordinatorenteam.
Hunderte Mitglieder helfen ehrenamtlich in unseren moderierten Foren.
Anbieter der Seite ist der gemeinnützige Verein "Vorhilfe.de e.V.".
Partnerseiten
Weitere Fächer:

Open Source FunktionenplotterFunkyPlot: Kostenloser und quelloffener Funktionenplotter für Linux und andere Betriebssysteme
Forum "Wahrscheinlichkeitstheorie" - Erwartungswert
Erwartungswert < Wahrscheinlichkeitstheorie < Stochastik < Hochschule < Mathe < Vorhilfe
Ansicht: [ geschachtelt ] | ^ Forum "Wahrscheinlichkeitstheorie"  | ^^ Alle Foren  | ^ Forenbaum  | Materialien

Erwartungswert: Frage (beantwortet)
Status: (Frage) beantwortet Status 
Datum: 12:29 Mi 31.10.2018
Autor: questionpeter

Aufgabe
Es seien [mm] X_1, X_2,.. [/mm] unabhängige Zufallsvariableb und es gelte [mm] X_k [/mm] Unif({0,...,k-1}). Für [mm] n\ge [/mm] 1 sei [mm] S_n=X_1+X_n, m_n=E(S_n), s_n^2=Var(S_n). [/mm] Für [mm] k\ge [/mm] 1 sei [mm] \tilde{X}_k=X_k-E(X_k). [/mm] Außerdem sei

[mm] S_n^*=\bruch{S_n-m_n}{s_n}=\bruch{\tilde{X}_1+\tilde{X}_2+...+\tilde{X}_n}{s_n} [/mm]

i) Bestimme [mm] m_n=E(S_n) [/mm] und [mm] s_n^2=Var(S_n) [/mm]

ii) Zeige

[mm] \bruch{1}{s_n^3}\sum_{k=1}^nE(\vert \tilde{X}_k\vert^3) \righrarrow [/mm] 0 für [mm] n\rightarrow [/mm] 0.

Hinwei: Es gilt [mm] \vert\tilde{X}_k\vert\le E(X_k) [/mm]

Hallo

i) [mm] Var(S_n)= [/mm] E( [mm] S_n-E(S_n))^2=E(X_1+...+X_n-(E(X_1+...+E(X_n))))^2=E(X_1-E(X_1)+...+X_n-E(X_n))^2=E(\tilde{X}_1+...+\tilde{X}_n)^2=(sum_k=1^n E(\tilde{X}_k))^2 [/mm]

[mm] E(S_n)= \sum_k=1^nE(X_k) [/mm]

Stimmt das soweit?

ii) Könnt ihr mir da einen Tipp geben

Danke im Voraus!


        
Bezug
Erwartungswert: Mitteilung
Status: (Mitteilung) Reaktion unnötig Status 
Datum: 14:06 Mi 31.10.2018
Autor: luis52

Moin questionpeter,

bitte stelle deine Aufgaben mit etwas mehr Sorgfalt ins Forum. U.a. macht $ [mm] \vert\tilde{X}_k\vert\le \operatorname{E}(X_k) [/mm] $ keinen Sinn: Links steht eine Zufallsvariable, rechts eine Zahl.

Bezug
                
Bezug
Erwartungswert: Mitteilung
Status: (Mitteilung) Reaktion unnötig Status 
Datum: 14:32 Mi 31.10.2018
Autor: questionpeter

Sorry. Der Hinweis steht so auf der Aufgabenstellung dabei.

Bezug
                        
Bezug
Erwartungswert: Mitteilung
Status: (Mitteilung) Reaktion unnötig Status 
Datum: 14:54 Mi 31.10.2018
Autor: luis52


> Sorry. Der Hinweis steht so auf der Aufgabenstellung dabei.

Kein Wunder, dass es mit Deutschland bergab geht, wenn an den Universitaeten solche Aufgaben gestellt werden. :-)

Und was ist mit $ [mm] \bruch{1}{s_n^3}\sum_{k=1}^nE(\vert \tilde{X}_k\vert^3) \righrarrow [/mm] $ 0 für $ [mm] n\rightarrow [/mm] $ 0?


Bezug
                                
Bezug
Erwartungswert: Mitteilung
Status: (Mitteilung) Reaktion unnötig Status 
Datum: 15:13 Mi 31.10.2018
Autor: questionpeter

Das ist mein Fehler. Aus lauter Hektik ist mir einen schreibfehler unterlaufen sodass der Pfeil nicht angezeigt werden konnte. Es sollte heißen:


$ [mm] \bruch{1}{s_n^3}\sum_{k=1}^nE(\vert \tilde{X}_k\vert^3) \rightarrow [/mm] 0 $ für n [mm] \rightarrow [/mm] 0

Danke für den Hinweis;).

Bezug
                                        
Bezug
Erwartungswert: Mitteilung
Status: (Mitteilung) Reaktion unnötig Status 
Datum: 15:15 Mi 31.10.2018
Autor: questionpeter

Für [mm] n\rightarrow \infty [/mm]

Bezug
                                        
Bezug
Erwartungswert: Mitteilung
Status: (Mitteilung) Reaktion unnötig Status 
Datum: 15:41 Mi 31.10.2018
Autor: luis52

Ist [mm] $S_n=X_1\red{+X_2+\dots}+X_n$? [/mm]

(Ich glaube, mir wird das hier zu anstrengend ...)

Bezug
                
Bezug
Erwartungswert: Mitteilung
Status: (Mitteilung) Reaktion unnötig Status 
Datum: 17:18 Mi 31.10.2018
Autor: Gonozal_IX

Hallo Luis,

auch wenn ich dir zur Formatierung der Aufgabe zustimme, bei der Aussage
>U.a. macht [mm]\vert\tilde{X}_k\vert\le \operatorname{E}(X_k)[/mm] keinen Sinn
kann ich das nicht… wieso sollte die Aussage
$X [mm] \le [/mm] c$ für X ZV und [mm] $c\in\IR$ [/mm] keinen Sinn machen?


Gruß,
Gono

Bezug
        
Bezug
Erwartungswert: Antwort
Status: (Antwort) fertig Status 
Datum: 17:28 Mi 31.10.2018
Autor: Gonozal_IX

Hiho,

also Luis hat es ja bereits angedeutet… dein Aufschrieb ist grausam weil unsorgfältig!
Gib dir zukünftig der Mühe, den Aufwand wälzt du sonst schlichtweg auf die Helfenden ab…


>  sei [mm] $S_n=X_1+X_n$ [/mm]

Da fehlen schlichtweg die Punkte, du meinst sicher [mm] $S_n [/mm] = [mm] X_1 [/mm] + [mm] \ldots [/mm] + [mm] X_n$ [/mm]

Weiterhin schreibe doch bitte lange Gleichungen in einer Matheumgebung… das kannst du dir auch fürs Studium eh angewöhnen.

> i) [mm]Var(S_n)=[/mm] E(
> [mm]S_n-E(S_n))^2=E(X_1+...+X_n-(E(X_1+...+E(X_n))))^2=E(X_1-E(X_1)+...+X_n-E(X_n))^2=E(\tilde{X}_1+...+\tilde{X}_n)^2=(\sum_{k=1}^n E(\tilde{X}_k))^2[/mm]
>  
> [mm]E(S_n)= \sum_{k=1}^nE(X_k)[/mm]
>  
> Stimmt das soweit?

Ja, und nun weitermachen!
Was ist [mm] $E(\tilde{X}_k)$? [/mm]
Was ist [mm] $E(X_k)$? [/mm]


> ii) Könnt ihr mir da einen Tipp geben

Hinweis verwenden, einsetzen.

Gruß,
Gono

Bezug
                
Bezug
Erwartungswert: Frage (beantwortet)
Status: (Frage) beantwortet Status 
Datum: 20:16 Mi 31.10.2018
Autor: questionpeter

Hallo,

wir wissen, dass [mm] X_k [/mm] unif({0,...,k-1}) ist und die Erwartungswert von einer Uniformen Verteilung lautet [mm] E(X)=\bruch{a+b}{2} [/mm] für [mm] a\le x\le [/mm] b, aber wir betrachten hier eine Menge {0,...,k-1}. Wie macht man das in diesem Fall?
Ist das [mm] E(X_k)=\bruch{k-1}{2}? [/mm]

[mm] \sum_{k=1}^nE(\tilde{X}_k)=s_n? [/mm]


zu ii) [mm] \bruch{1}{s_n^3}\sum_{k=1}^nE(\vert X_k\vert^3)\le \bruch{1}{s_n^3}\sum_{k=1}^nE(E(X_k)^3)=\bruch{1}{s_n^3}\sum_{k=1}^nE(X_k)^3 [/mm]

Bezug
                        
Bezug
Erwartungswert: Antwort
Status: (Antwort) fertig Status 
Datum: 21:41 Mi 31.10.2018
Autor: Gonozal_IX

Hiho,

> wir wissen, dass [mm]X_k[/mm] unif({0,...,k-1}) ist und die
> Erwartungswert von einer Uniformen Verteilung lautet
> [mm]E(X)=\bruch{a+b}{2}[/mm] für [mm]a\le x\le[/mm] b

Du wirfst hier diskrete und stetige Gleichverteilung in einen Topf. Ist das sinnvoll?


> aber wir betrachten hier eine Menge {0,...,k-1}. Wie macht man das in diesem Fall?

Na berechne doch mal den Erwartungswert ganz stupide!!

>  Ist das [mm]E(X_k)=\bruch{k-1}{2}?[/mm]

Ist es das? Wenn du dir unsicher bist: AUSRECHNEN!
Das ist nun wirklich nicht zu viel verlangt.

> [mm]\sum_{k=1}^nE(\tilde{X}_k)=s_n?[/mm]

Wieso sollte das gelten?

>
> zu ii) [mm]\bruch{1}{s_n^3}\sum_{k=1}^nE(\vert X_k\vert^3)\le \bruch{1}{s_n^3}\sum_{k=1}^nE(E(X_k)^3)=\bruch{1}{s_n^3}\sum_{k=1}^nE(X_k)^3[/mm]

Und wo bleibt das Einsetzen?
Was ist nun [mm] $E(X_k)$? [/mm]
Siehe oben: Ausrechnen!
Und [mm] s_n [/mm] wirst du wohl auch brauchen…

Gruß,
Gono

Bezug
                                
Bezug
Erwartungswert: Frage (beantwortet)
Status: (Frage) beantwortet Status 
Datum: 09:21 Do 01.11.2018
Autor: questionpeter

[mm] s_n^2=Var(S_n) [/mm] und [mm] Var(S_n)=(\sum_{k=1}^nE(\tilde{X}_k))^2. [/mm] Daher
[mm] s_n=\sum_{k=1}^nE(\tilde{X}_k) [/mm]

und [mm] E(\tilde{X_k})=E(X_k-E(X_k))=E(X_k)-E(X_k)=0? [/mm] (viel bringt mir diese Aussage jetzt nicht.)


[mm] E(X_k)=\int_{0}^{k-1}xf(x)dx=\bruch{1}{k-1}\int_{0}^{k-1}xdx=\bruch{1}{k-1}\bigg[\bruch{1}{2}x^2\bigg]_{0}^{k-1}=\bruch{k-1}{2} [/mm]

also [mm] E(X_k)=\sum_{k=1}^nE(X_k)=\sum_{k=1}^n\bruch{k-1}{2}=\bruch{1}{2}\sum_{k=1}^n(k-1)=\bruch{1}{2}\sum_{k=0}^{n-1}k=\bruch{(k-1)k}{4}??? [/mm]

[mm] \bruch{1}{s_n^3}\sum_{k=1}^nE(\vert X_k\vert^3)\le \bruch{1}{s_n^3}\sum_{k=1}^nE(E(X_k)^3)=\bruch{1}{s_n^3}\sum_{k=1}^nE(X_k)^3=\bruch{1}{s_n^3}\sum_{k=1}^n \bigg(\bruch{(k-1)k}{4}\bigg)^3 [/mm]

Stimmt das bis hier hin?

Bezug
                                        
Bezug
Erwartungswert: Antwort
Status: (Antwort) fertig Status 
Datum: 09:32 Do 01.11.2018
Autor: Gonozal_IX

Hiho,

> [mm]s_n^2=Var(S_n)[/mm] und [mm]Var(S_n)=(\sum_{k=1}^nE(\tilde{X}_k))^2.[/mm]

Das stimmt aber gar nicht… wo wir wieder beim sauberen Aufschrieb sind.

Deine Klammersetzung ist ein graus und lässt dich Fehler machen!
Es ist (diesmal mit sauberer, korrekter Klammersetzung):

$ [mm] \text{Var}(S_n)= E\left[\left(S_n-E[S_n]\right)^2\right]=E\left[\left(X_1+\ldots+X_n-\left(E[X_1]+\ldots+E[X_n]\right)\right)^2\right]=E\left[\left(X_1-E[X_1]+\ldots+X_n-E[X_n]\right)^2\right]=E\left[\left(\tilde{X}_1+\ldots+\tilde{X}_n\right)^2\right]= \sum_{k=1}^n E\left[\left(\tilde{X}_k\right)^2\right] [/mm] $

Und jetzt ist mir auch klar, dass du vermutlich keine Ahnung hast, warum das letzte Gleichheitszeichen gilt!
Daher die Frage: Warum gilt das letzte Gleichheitszeichen?


>  
> und [mm]E(\tilde{X_k})=E(X_k-E(X_k))=E(X_k)-E(X_k)=0?[/mm]

Das stimmt zwar, aber dann hätten dich deine Umformungen stutzig machen müssen! Dann wäre nach deiner Berechnung ja [mm] $s_n [/mm] = 0$ und damit [mm] $\text{Var}(S_n) [/mm] = 0$.
Kann das sein? Welche Prozesse haben eine Varianz von Null?


> [mm]E(X_k)=\int_{0}^{k-1}xf(x)dx=\bruch{1}{k-1}\int_{0}^{k-1}xdx=\bruch{1}{k-1}\bigg[\bruch{1}{2}x^2\bigg]_{0}^{k-1}=\bruch{k-1}{2}[/mm]

Das ist doch, gelinde gesagt, totaler Blödsinn.
Die [mm] X_k [/mm] sind diskrete Zufallsvariablen.
Was du oben berechnest ist der Erwartungswert einer stetigen Gleichverteilung.

Wie berechnet sich der Erwartungswert einer diskreten Zufallsvariable?

Gruß,
Gono

Bezug
                                                
Bezug
Erwartungswert: Frage (beantwortet)
Status: (Frage) beantwortet Status 
Datum: 10:57 Do 01.11.2018
Autor: questionpeter

Gleichheit gilt, weil Erwartungswert linear ist.

[mm] E(X_k)=\bruch{1}{k}\sum_{l=0}^{k-1}l=\bruch{1}{k}\sum_{k=1}^{k}l=\bruch{(k+1)k}{2k}? [/mm]

Bezug
                                                        
Bezug
Erwartungswert: Antwort
Status: (Antwort) fertig Status 
Datum: 07:38 Fr 02.11.2018
Autor: Gonozal_IX

Hiho,

> Gleichheit gilt, weil Erwartungswert linear ist.

Falsch.

Da steht ein Quadrat im Erwartungswert… die Quadratfunktion ist nicht linear, wie willst du da mit Linearität argumentieren.

Machen wir es mal zuerst für $n=2$.

Links steht: [mm] $E\left[\left(\tilde{X_1} + \tilde{X_2}\right)^2\right]$ [/mm]
Rechts steht: [mm] $E\left[\tilde{X_1}^2\right] [/mm] + [mm] E\left[\tilde{X_2}^2\right]$ [/mm]

In der Schule lernt man, dass im Allgemienen [mm] $(x_1 [/mm] + [mm] x_2)^2 \not= x_1^2 [/mm] + [mm] x_2^2$ [/mm]
Linearität allein kann es also nicht sein.

Begründe nun, warum [mm] $E\left[\left(\tilde{X_1} + \tilde{X_2}\right)^2\right] [/mm] = [mm] E\left[\tilde{X_1}^2\right] [/mm] + [mm] E\left[\tilde{X_2}^2\right]$ [/mm] trotzdem gilt.

Wann gilt denn obige Gleichung?
Tipp: Löse die linke Seite mal mit Hilfe der binomischen Formel auf…

> [mm]E(X_k)=\bruch{1}{k}\sum_{l=0}^{k-1}l=\bruch{1}{k}\sum_{k=1}^{k}l=\bruch{(k+1)k}{2k}?[/mm]

Die Idee ist richtig. Wo kommt das [mm] $\frac{1}{k}$ [/mm] her? Deine Indexverschiebung ist falsch, sonst wärst du auf die richtige Lösung gekommen.
Es ist offensichtlich   [mm] $\sum_{l=0}^{k-1}l [/mm] = [mm] \sum_{l=1}^{k-1}l$ [/mm]

Gruß,
Gono


Bezug
                                                                
Bezug
Erwartungswert: Frage (beantwortet)
Status: (Frage) beantwortet Status 
Datum: 09:15 Fr 02.11.2018
Autor: questionpeter

Hallo,

Es fällt mir nun wie Schuppen von den Augen. Danke.

Aufgrund [mm] \tilde{X_k}=X_k-E(X_k), [/mm] denn wir haben [mm] E(\tilde{X_k})=E(X_k-E(X_k))=E(X_k)-E(X_k)=0 [/mm] und dann bleiben am Ende genau alle EW in Quadrat übrig.

Bezug
                                                                        
Bezug
Erwartungswert: Antwort
Status: (Antwort) fertig Status 
Datum: 09:30 Fr 02.11.2018
Autor: Gonozal_IX

Hiho,

> Es fällt mir nun wie Schuppen von den Augen. Danke.

Das ist schön…

> Aufgrund [mm]\tilde{X_k}=X_k-E(X_k),[/mm] denn wir haben
> [mm]E(\tilde{X_k})=E(X_k-E(X_k))=E(X_k)-E(X_k)=0[/mm] und dann
> bleiben am Ende genau alle EW in Quadrat übrig.  

Da haben wir aber noch ein paar Lagen Schuppen übrig…
Du wirfst immer nur was hin, was in keinem Zusammenhang mit dem von mir gefragten steht… zumindest ist obige Begründung keine für die Gleichheit.

Fang mal bitte an mit
$ [mm] E\left[\left(\tilde{X_1} + \tilde{X_2}\right)^2\right] [/mm] $
und forme so lange um, bis da
$ [mm] E\left[\tilde{X_1}^2\right] [/mm] + [mm] E\left[\tilde{X_2}^2\right] [/mm] $
steht.
Und begründe jedes einzelne Gleichheitszeichen!

Das kann doch nicht so schwer sein… (und dann wirst du feststellen, dass deine Begründung allein nicht ausreicht)

PS: Dir fehlen echt Grundlagen… und sauberes Arbeiten. Du solltest das dringend nachholen!

Gruß,
Gono

Bezug
Ansicht: [ geschachtelt ] | ^ Forum "Wahrscheinlichkeitstheorie"  | ^^ Alle Foren  | ^ Forenbaum  | Materialien


^ Seitenanfang ^
ev.vorhilfe.de
[ Startseite | Mitglieder | Impressum ]